Sunteți pe pagina 1din 45

Block 3 1. A 60-year-old white woman comes to the office because of a 6- to 12-month history of weakness.

She initially noted the weakness in her right hand, and it has now become generalized and is associated with muscle cramps. She is not taking any medications. Past medical history is unremarkable. On physical examination there is marked muscle wasting of the hands and arms bilaterally (left greater than right). There is less muscle wasting in the legs. Sensation to touch remains intact. Muscle fasciculation of the arms and hands is noted and deep tendon reflexes are decreased. Speech and mentation are intact. Which of the following studies is most likely to be abnormal in this patient? A) Edrophonium chloride test B) Electromyography C) Serum protein electrophoresis D) Urine lead levels E) Urine mercury levels 2. A 23-year-old primigravid woman comes to the office to begin prenatal care. She and her husband had been having unprotected sexual intercourse for 8 months prior to becoming pregnant. The patient is excited about the pregnancy. She is at 10 weeks' gestation based on the date of her last menstrual period. Medical history is significant for a 6-year history of intermittent left-sided seizures that began following a motor vehicle accident during high school. The patient has taken valproic acid therapy for the past 4 years with good control of the seizures. Vital signs today are temperature 36.7C (98.0F), pulse 80/min, respirations 20/min, and blood pressure 110/60 mm Hg. Which of the following is the most appropriate modification to the patient's pharmacotherapy? A) Discontinue the valproic acid until after pregnancy B) Switch the valproic acid to diazepam C) Switch the valproic acid to phenobarbital D) Switch the valproic acid to phenytoin E) No modification is indicated 3. A 32-year-old secretary comes to the office for a periodic health evaluation and Pap smear. She says, "I've been doing fine except that for the past 2 months my right hand sometimes has been tingling and feels numb, especially at night. I'm always dropping things, too." She takes oral contraceptive pills and thyroid replacement medication for hypothyroidism. She is right-hand dominant. She has no history of acute trauma. On physical examination, she has numbness and tingling in the thumb and first two digits of her right hand after 15 seconds of a wrist flexion test. The remainder of the examination is normal. Which of the following studies is most likely to establish a diagnosis? A) Cervical spine x-ray films B) MRI of the brain C) Nerve conduction studies

D) Serum thyroid-stimulating hormone (TSH) concentration E) X-ray film of the right hand and wrist 4. A 48-year-old Iranian American man comes to the office because of chronic dry cough, weight loss and intermittent temperatures to 38.3C (101.0F) for the past 2 months. He has lost 3.5 kg (8 lb) during this time. Today vital signs are: temperature 38.3C (100.9F), pulse 90/min, respirations 18/min and blood pressure 120/78 mm Hg. Auscultation of the lungs discloses fine crackles throughout the lung fields. The remainder of the physical examination is normal. Chest x-ray film is shown. Therapy with isoniazid, rifampin, pyrazinamide and ethambutol is begun. Your office staff ask you whether they should be evaluated for exposure to tuberculosis. Which of the following is the most appropriate next step? A) Do nothing for office staff because the patient is unlikely to be infectious B) Monitor the staff during the next few weeks for development of cough or fever, and base further treatment on this information C) Obtain chest x-ray films in 4 weeks of each staff member who came into contact with the patient D) Place a 5-TU PPD skin test in 4 weeks on each staff member who came into contact with the patient E) Prescribe prophylactic isoniazid therapy to all office staff 5. A 67-year-old man comes to the office for an initial visit. He says his daughter, who is a patient in your practice, "made me come" because of a lesion in his mouth under his tongue. He is a farmer from the South who is visiting his daughter, and he is not concerned about the lesion. He says, "It has been present for a couple of months, and although I can feel it with the tip of my tongue and it is slightly sore, it doesn't give me any trouble." He has always been healthy and takes no medication. He drinks alcohol in small amounts on weekends, and he uses tobacco in a variety of forms. On physical examination, a 2H2-cm, raised, roughened gray lesion on the oral mucosa of the left side of the floor of the mouth is noted extending to the base of the tongue. You decide the lesion is highly suggestive of a malignancy and that a biopsy should be obtained. Based on the patient's history and incidence of lesions in this location, the lesion is most likely which of the following? A) Adenocarcinoma B) Basal cell carcinoma C) Leukoplakia D) Lymphoma E) Squamous cell carcinoma 6. A 52-year-old African-American man returns to the health center for a follow-up visit 1 week after he was discharged from the hospital after being diagnosed with acute pneumonia. Discharge medications included inhaled bronchodilators, oral antibiotics and a tapering course of oral corticosteroids. The antibiotic and corticosteroid therapies were completed yesterday. He has no previous hospital admissions for respiratory disease. He

has a nonproductive, chronic, daily cough. He smoked one to two packs of cigarettes per day since age 13 years, but he has not smoked since admission to the hospital. Vital signs today are: temperature 36.8C (98.2F), pulse 92/min, respirations 10/min and blood pressure 120/80 mm Hg. On physical examination he appears well. There are markedly diminished, but clear, breath sounds with no wheezing or rhonchi; there is a prolonged expiratory phase. There is no digital clubbing. Which of the following is the most likeley diagnosis? A) Asthma B) Bronchiectasis C) Chronic bronchitis D) Emphysema E) Pulmonary fibrosis The following vignette applies to the next 3 items. Patient History Sex: Male Age: 53 years Sociodemographic information: African-American high school teacher, married, no tobacco use, history of heavy alcohol use but not for the past 10 years Medical history: Hypertension; peptic ulcer disease 15 years ago; pneumonia-related hospitalization 15 years ago; no known drug allergies Family history: Mother died of renal failure due to diabetes and hypertension at age 69 years; father died of heart failure and hypertension; two sisters have hypertension Current medications/drug information: One aspirin tablet daily Today's Visit Height: 185 cm (6 ft 1 in) Weight: 87 kg (192 lb) Vital signs: Temperature 36.8C (98.2F) Respirations 16/min Pulse 88/min, regular Blood pressure 150/95 mm Hg

Physical examination: Blood pressure is equal in both arms, fundi show arteriolar narrowing; remainder of physical examination is normal Laboratory studies: Electrocardiogram shows normal sinus rhythm and left ventricular hypertrophy The patient whose chart is shown comes to the office for the first time because he has been transferred to your managed care organization. He had been taking a blood pressure medication but explains that he ran out of it 8 months ago. He did not make an appointment until now because, he says, "I just never got to it." His wife persuaded him to seek care now. He tells you that he requested his previous records be sent to you but they have not arrived. Item 1 of 3 7. Which of the following is the most appropriate pharmacotherapy for this patient? A) Clonidine B) Enalapril C) Hydrochlorothiazide D) Nifedipine E) Terazosin tem 2 of 3 8. The patient returns to the office in 1 week for a follow-up visit. He says he has a cold with cough, nasal congenstion and a scratchy throat. Blood pressure in now 140/85 mm Hg. On physical examination lungs are clear and there is no peripheral edema. Regarding over-the-counter medications for this patient's cold, he should be cautioned to avoid which of the following? A) Chlorpheniramine B) Dextromethorphan C) Guaifenesin D) Oxymetazoline E) Pseudoephedrine Item 3 of 3 9. Regarding his blood pressure, which of the following is the most appropriate management? A) Add a second antihypertensive medication to the regimen B) Add potassium chloride supplements to the medication

C) Change the antihypertensive medication D) Continue with the current regimen E) Increase the dose of the prescribed medication 10. A family of two adults and two school-aged children comes to the office because each family member has had intermittent nausea and diarrhea during the past 4 to 6 weeks. They are at increased risk for giardiasis if which of the following is true? A) A close family friend has AIDS B) Their water supply is a shallow well C) They frequently consume uncooked vegetables D) They recently adopted a new pet dog E) They recently returned from a trip to Mexico 11. A 61-year-old Dutch businessman comes to the office to discuss the results of screening flexible sigmoidoscopy done 1 week ago. He has hypertension and hypercholesterolemia, for which he takes hydrochlorothiazide and pravastatin. His grandmother died of colon cancer, but no other family members have had colon cancer or polyps. During the sigmoidoscopy, the colon was well visualized to 60 cm and a 1-cm polyp was removed. On pathologic examination, the polyp was found to be adenomatous. Which of the following is the most appropriate diagnostic study at this time? A) Colonoscopy B) Determination of serum -fetoprotein concentration C) Determination of serum carcinoembryonic antigen concentration D) Double-contrast lower gastrointestinal barium study E) No further study is indicated 12. A 52-year-old woman is brought to the office by her husband for evaluation of increasing tearfulness, restlessness, difficulty sleeping and weight loss for 1 month. During the visit, the patient appears restless and upset. She acknowledges her husband's account of her symptoms, but she does not volunteer any additional information. They have been married for 15 years and have a good relationship. Both the patient and her husband are teachers and they have a healthy 13-year-old son. She was previously healthy. Physical examination today is normal. The most appropriate initial management of her symptoms is to do which of the following? A) Prescribe haloperidol B) Prescribe lithium C) Prescribe lorazepam D) Prescribe sertraline E) Refer them for couples therapy 13. A 14-year-old white girl is brought to the health center by her mother. The mother is in tears and states, "My daughter has been suspended from school for cursing at a teacher in class. Last night she came home 2 hours after her curfew. When I confronted her she

yelled at me and pushed me into the wall. I don't know what is wrong with her." The girl states, "There is nothing wrong with me. I just want to be with my friends and do what I want. I don't feel I need a curfew. I didn't do anything wrong at school. The teacher mouthed off at me. She should have been suspended instead of me." The girl has been your patient since age 10 years when the family moved from another state. She has a history of allergies to mold and dust for which she uses loratadine as needed. She takes no other medications. Her menses began at age 12 years. Vital signs now are: temperature 37.0C (98.6F), pulse 90/min, respirations 12/min and blood pressure 106/60 mm Hg. Physical examination is normal. Which of the following is the most appropriate opening question to the patient when you are speaking alone with her? A) "Are you using drugs or alcohol?" B) "How do you want your parents to react to your violating their rules?" C) "What seems to be bothering you the most?" D) "Why aren't you showing your parents any respect?" E) "Why did you push your mother?" 14. A 7-year-old white boy is brought to the health center by his mother because of stomach pain for the past 3 months. The mother says that the pain typically comes on soon after waking and is not relieved by antacids. She says his symptoms never occur late in the day, but only in the mornings. There has been no vomiting, diarrhea or weight loss. The child has been generally healthy and is up-to-date with vaccinations. He lives with both parents. His height and weight are at the 75th percentile. Physical examination, including developmental assessment, is normal. Specific additional history should be obtained regarding which of the following? A) Days of the week in which the patient is most symptomatic B) Family history of milk allergy C) Quantity of food intake D) Recent travel history E) The source of water at home 15. A 49-year-old man, who is accompanied by his son, comes to the office for the first time for follow-up of mild depression. He recently moved to the area and he tells you that his previous physician initiated fluoxetine therapy 4 weeks ago. He has a history of stage I malignant melanoma, which was treated with wide, local excision 2 years ago. He mentions that he was adopted as a young child and his family history is unknown. The patient is friendly and engaging, seems to be full of energy and calls you by your first name. He reports that he has been unusually productive at work recently and is able to "get by" on 1 to 2 hours of sleep nightly. His son says, "Dad isn't acting like himself. He talks about weird things and he's become religious in a creepy way." On physical examination the patient is well developed and well nourished. Vital signs are: temperature 37.0C (98.6F), pulse 95/min, respirations 14/min and blood pressure 135/90 mm Hg. There is a mild, nonpainful increase in tactile sensory acuity. He says, "My skin is really sensitive." The remainder of the physical examination, including neurologic examination, is normal. Which of the following is the most likely precipitant

of this episode? A) Family conflict B) Fluoxetine C) Recurrence of melanoma in his central nervous system D) Stress E) Unknown; this cannot be determined 16. A 62-year-old man returns to the office to discuss findings of x-ray films of the lumbosacral spine obtained 2 months ago following an office visit for evaluation of chronic low back pain. Physical examination at that time was normal. He was instructed to take aspirin as needed for pain and to await notification from the office regarding when he should return to discuss the x-ray films. The office staff failed to schedule a return visit for the patient; today's visit was scheduled by the patient. The patient says, "I've been extremely anxious about the potential results of the x-ray film, and I'm angry that nobody from the office called me to schedule a follow-up visit." The radiologist's report accompanying the x-ray film notes the presence of a lytic lesion at the L1 vertebra consistent with metastatic cancer and the recommendation that a CT scan be obtained. In addition to apologizing to the patient for neglecting to contact him to schedule follow-up, which of the following is the most appropriate approach to informing the patient of the xray film findings? A) Inform him of the findings and emphasize that the responsibility to schedule a followup appointment was his B) Inform him of the findings and emphasize the need for further evaluation C) Inform him of the findings and tell him that the radiologist is as much to blame for not informing him of the urgency of his condition D) Inform him that the findings are nonspecific and that further studies are needed to identify a diagnosis E) Tell him that the radiologist's report is likely overstated and that additional x-ray films will need to be obtained 17. A 21-year-old woman who is a varsity basketball player comes to the student health center because of left knee pain for the past month. She says the pain has progressively worsened and has not been relieved with ibuprofen. The pain occurs when she descends stairs or is in class for long periods. She says, "My knee pops a lot." Physical examination today shows knee crepitus, and weakness and atrophy of the vastus medialis muscle. Which of the following is the most appropriate treatment at this time? A) Anterior cruciate ligament reconstruction B) Arthroscopic meniscectomy C) Immobilization of the knee D) Nonweight-bearing on crutches for 2 weeks E) Quadriceps-strengthening exercises 18. A 39-year-old African-American woman is brought to the health center at 7:00 PM

by her children because of weakness and numbness in her right leg. She also says, "I have an odd feeling in my left leg." The symptoms started several days ago and gradually have increased. She also experienced low back pain about 10 days ago, which responded until recently to oral morphine and ibuprofen prescribed by her primary care physician, whom she could not reach tonight. She has had no problems with bowel or bladder function. She was treated 4 months ago with mastectomy, chemotherapy and radiation therapy for infiltrating ductal carcinoma of the left breast. Neurologic examination today discloses weakness of all muscle groups in the right leg and thigh, with normal strength in the left leg. There is marked decrease in sensation to pinprick and temperature up to the area of the inguinal ligament on the right, and decreased perception of position and vibration sense in the left lower extremity. The patellar and Achilles reflexes are hyperactive on the right and those on the left are normal. Rectal sphincter tone is normal. Diagnostic studies show metastatic disease to L2 with minimum invasion of the spinal canal. Which of the following is the most appropriate method to control her pain? A) Chemotherapy B) Dorsal rhizotomy C) Long-acting oral morphine therapy D) Radiation therapy E) Surgical decompression 19. A 55-year-old man comes to the office because of a 1-month history of right knee pain. He reports no recent trauma to his knee but says that he has recently increased his weekly running distance from 12 miles to 30 miles. On physical examination he has aching knee pain with squatting. The rest of the physical examination is normal. Which of the following factors in the patient's history , if present, would indicate the patellofemoral joint as the source of the pain? A) Associated thigh pain B) Buckling of the knee C) Increased pain with stair climbing D) Locking of the knee E) Prominent knee swelling 20. A 24-year-old professional gardener comes to the office because of a generalized, maculopapular, itchy rash that developed a few days ago. The itching is keeping him awake at night. Three weeks ago he began treatment for a seizure disorder with 400 mg of phenytoin, daily. Physical examination now shows a generalized maculopapular rash with excoriations. The most appropriate management at this time is discontinuation of the drug and addition of which of the following? A) Oral hydroxyzine B) Oral prednisone C) Topical betamethasone D) Topical emollients E) Topical hydrocortisone

21. A 58-year-old white man comes to the office because of a mass in his neck for the past 2 months. He says, "My voice is bad and sometimes I can't catch my breath." He is a construction worker and has a long history of alcohol abuse and cigarette smoking. Vital signs are normal. Physical examination shows a large mass in the floor of the mouth, extending to the tonsillar fossa. A 4H5-cm mass is noted in the right anterior triangle of the neck. The greatest immediate risk to the patient is which of the following? A) Airway obstruction B) Aspiration C) Bleeding D) Sepsis E) Stroke 22. A 15-year-old girl comes to the office because of a 2-day history of an itchy rash on her right ankle. The rash occurred 1 day after she took a school field trip to a local park while wearing a new pair of sandals. She has moderate to severe asthma, for which she takes zafirlukast, salmeterol, and albuterol by metered-dose inhaler as needed. She has been tapering off oral prednisone for the past 2 months. Vital signs are temperature 37.5C (99.5F), pulse 86/min, respirations 16/min, and blood pressure 125/80 mm Hg. Auscultation of the lungs discloses mild, diffuse expiratory wheezing. Her right ankle is shown. Which of the following is the most likely diagnosis? A) Chigger infestation B) Contact dermatitis C) Herpes zoster D) Poison ivy dermatitis E) Scabies The following vignette applies to the next 2 items. A 29-year-old truck driver comes to the health center for advice regarding his serum cholesterol concentration, which was determined recently at a shopping mall health fair. He had the test at the request of his wife, who is pregnant with their first child. The test result read "high," with advice to see a physician. The patient has a family history of premature coronary artery disease. Two uncles had myocardial infarctions (MI), one at age 41 years and the other at age 45 years. His father died of an MI at age 47 years. He tells you that his 34-year-old brother takes "some type of cholesterol pill." The patient has no significant medical history. He denies smoking cigarettes but admits to drinking a few beers in the evenings at home. Two to three times per week he meets his friends at a local tavern before driving home. He is currently taking no medications, and his physical examination is normal except for mild obesity. A fasting serum lipid profile was done 2 days ago at the suggestion of the nurse who took his history by phone. Results are available today and show:

Serum Cholesterol Total 171 mg/dL HDL 29 mg/dL LDL 117 mg/dL Triglycerides 156 mg/dL Item 1 of 2 23. To evaluate his risk for cardiac disease, in addition to his serum lipid concentrations it is appropriate to obtain which of the following serum laboratory studies? A) Angiotensin II concentration B) Apolipoprotein B concentration C) Endothelin concentration D) Homocysteine concentration E) Lipoprotein lipase concentration

(Hypoalphalipoproteinemia)

Item 2 of 2 24. With respect to his alcohol intake, it is most appropriate to do which of the following? A) Determine if his uncles and father drank and whether it affected their lipid levels B) Encourage him to continue drinking in moderation given his lipid analysis C) Investigate the risk of his drinking while driving his truck D) Recommend that he switch from beer to red wine E) Recommend that he switch to a low-calorie beer 25. A 50-year-old African-American woman returns to the office for follow-up of diabetes mellitus, which has been treated with diet; extended-release glipizide, 10 mg daily; and metformin, 500 mg twice a day. She says, "I do the best I can adhering to my diet." She tests her blood glucose concentration daily. For the past month her fasting blood glucose concentrations have averaged 170 mg/dL. Hemoglobin A1c 1 week ago was 8.4%. The patient is 167.5 cm (5 ft 6 in) tall and weighs 86 kg (190 lb). Which of the following is the most appropriate change in therapy? A) Add chlorpropamide B) Add insulin C) Increase the metformin dosage D) Stop the glipizide and metformin and start insulin

E) No change is indicated 26. A 36-year-old African-American woman with hypertension comes to the office because of a 3-day history of intermittent, painless, gross hematuria. Symptoms started spontaneously, and she has not had fever, chills, flank pain or dysuria. Her only medication is hydrochlorothiazide. Vital signs are: temperature 37.0C (98.6F), pulse 72/min and blood pressure 128/84 mm Hg. Physical examination is normal; there is no costovertebral angle tenderness. Laboratory studies show: Serum Urine Electrolytes Normal Protein 3+ Glucose 78 mg/dL Blood Positive BUN 8 mg/dL Microscopic RBCs too numerous to count Blood No WBCs, bacteria or casts CBC Normal Nitrite Negative Partial thromboplastin time Normal Urine culture Pending Platelet count 268,000/mm3 Prothrombin time Normal Which of the following is the most appropriate next step? A) Cystoscopy B) Determination of protein excretion in a 24-hour urine sample C) Initiation of antibiotic therapy D) Renal biopsy E) Transvaginal ultrasonography of the uterus

The following vignette applies to the next 2 items. A 15-year-old girl is brought to the office for the first time by her mother, who has been your patient for many years. The mother is concerned because her daughter, who had menarche at age 11 years, has not had a period in the past 6 months. Both mother and daughter have kept a record of the girl's periods. The first three periods were irregularly spaced and not painful. Subsequent periods were regularly spaced and were accompanied by severe cramping on the first day. The cramping subsided 1 year ago. The mother insists that you interview her and her daughter together. She says that her daughter is not sexually active. She tells you that her daughter gets straight A grades and that she is very active in sports. The girl is also a cheerleader and is taking a night course for college credit at the local community college through the school's gifted child program. The mother tells you that her daughter has a good appetite for healthy food and does not eat junk food. She asks you what is causing her daughter's amenorrhea. Item 1 of 2 27. Which of the following is the most appropriate initial response? A) "In young women, missing a few periods is not unusual, and we should wait a while longer before becoming concerned." B) "It is a well-known fact that the most likely cause of amenorrhea in young women is pregnancy. How can you be certain she has not been sexually active?" C) "Many things can cause missed periods. After I examine your daughter, I may be able to provide more insight into the problem." D) "Over-involvement in some athletic activities can cause menstrual periods to cease." E) "Your daughter is involved in a lot of activities and may be under stress. That can certainly be a factor in causing cessation of periods." Item 2 of 2 28. After responding to the mother's question, you ask the mother to wait outside while you speak privately with the girl and do a physical examination. No additional information is obtained. Vital signs are: temperature 37.0C (98.6F), pulse 110/min and blood pressure 90/50 mm Hg. Height is 165 cm (5ft 5 in) and weight is 49 kg (105 lb). Physical examination shows some eroding of the enamel on the girl's rear teeth. Sexual maturation is complete. Abdomen is soft and nontender. Which of the following is the most likely diagnosis? A) Athletic amenorrhea B) Bulimia C) Hypothyroidism D) Pregnancy E) Stress-induced amenorrhea

29. A 39-year-old white homemaker comes to the office because of abnormal vaginal bleeding. You provide routine medical care for her and her family. She says her menstrual periods always have been regular and that her last normal regular period was about 10 weeks ago. She has had spotting for 1 week. She tells you that she and her husband use "withdrawal" as their birth control method. Urine pregnancy test is positive. Vital signs now are: temperature 37.4C (99.3F), pulse 88/min and blood pressure 110/72 mm Hg. Pelvic examination discloses a small amount of blood in the vagina. Cervix is closed. Uterus is soft, nontender and anteverted. Adnexae are nontender. Additional history reveals that she had a first trimester elective abortion at age 20 years and she has genital herpes that recurs about twice each year. Her husband is age 40 years and has Crohn disease. Her children are ages 7 and 9 years. The 7-year-old was born by cesarean delivery at 32 weeks' gestation after placental abruption. This couple's greatest risk factor for miscarriage is which of the following? A) History of abruptio placenta B) History of genital herpes C) History of premature delivery D) Maternal age E) Paternal history of auto-immune disease 30. A 30-year-old woman who is 28 weeks pregnant with her first child comes to the office for a routine prenatal visit. She has been a strict vegetarian for many years. At each prenatal visit she has insisted that she is eating well. However, she has gained only 0.9 kg (2 lb) since her initial prepregnancy weight of 54 kg (120 lb); she is 165 cm (5 ft 5 in) tall. She says that she has been taking her prenatal vitamins and iron as instructed. She has a regular exercise routine but has decreased this activity significantly during the past 4 weeks because, she says, "I've been too tired." Pulse is 90/min and blood pressure is 110/60 mm Hg. Physical examination shows a reddened, malar rash. Laboratory studies at this visit show: Hematocrit 25% Hemoglobin 7.5 g/dL WBC 3500/mm3 MCV 105 m3 Platelet count 89,000/mm3 Which of the following is most appropriate to prescribe orally at this time? A) Corticosteroid therapy B) Vitamin Bl2 (cyanocobalamin) supplementation C) Folate supplementation D) Iron supplementation E) Vitamin E (tocopherol) supplementation 31. A 36-year-old African-American woman with hypertension comes to the office because of a 3-day history of dysuria, hematuria, and urinary frequency and urgency. Symptoms started the day after she returned from a weekend visit with her boyfriend in

another city. She has no prior history of urinary tract infections and she denies fever, chills, flank pain, nausea or vomiting. Her only medication is hydrochlorothiazide. Vital signs at today's visit are: temperature 37.6C (99.6F), pulse 90/min and blood pressure 126/84 mm Hg. Physical examination is normal. Laboratory studies show: Urine Blood Positive WBC 15-20/hpf RBC 15-20/hpf Bacteria Moderate Casts Negative Nitrate Positive Protein 1+ Which of the following is the most appropriate next step? A) Increase oral intake of fluids and order a urine culture B) Order cystoscopy C) Order intravenous urography D) Repeat the urinalysis in 3 to 4 days E) Start antibiotic therapy 32. A 54-year-old white man comes to the office because of increasingly severe nosebleeds for the past 10 days. He says initially his nosebleeds resolved spontaneously, but over the past week they have become more frequent and have had a heavier flow. He says his nose has been bleeding almost constantly for the past 48 hours. There is no history of trauma or previous nosebleeds. His only medication is quinidine, which was started 6 weeks ago for restless legs. Medical history is otherwise unremarkable and he takes no other medications. Vital signs are: temperature 36.5C (97.7F), pulse 75/min, respirations 18/min, and blood pressure 132/85 mm Hg. His nose is packed with tissue paper and he has blood stains on his shirt. Examination of the nose shows no active bleeding or nasal lesions, but there is dried blood on his nares. Petechiae are noted inside the oral cavity. Examination of the skin shows scattered ecchymoses over the lower extremities. The remainder of the physical examination is noncontributory. Results of laboratory studies show: Serum Blood ALT 32 U/L Hematocrit 37% AST 43 U/L Hemoglobin 11.7 g/dL WBC 6500/mm3 PTT 22 seconds INR 1.0 Platelet count 32,500/mm3 Which of the following is the most appropriate initial management? A) Bone marrow biopsy B) Determination of bleeding time

C) Discontinuation of quinidine D) Platelet transfusion E) Vasopressin therapy 33. At an annual company examination, a 40-year-old man has an elevated mean corpuscular volume. Hemoglobin concentration is normal. This finding is most suggestive of which of the following? A) Alcohol abuse B) Hemosiderosis C) Hepatitis B infection D) Schistosomiasis E) Vitamin B6 (pyridoxine) deficiency 34. A 15-year-old white girl is brought to the health center by her mother for a periodic health evaluation and refill of tetracycline for acne. Upon entering the examination room, the patient's mother asks to speak alone with you. In private, she says, "My daughter has been like a different person for the past 4 months. She used to be on the honor roll and is now failing school. She received detention six times for truancy and for talking back to teachers. She argues with her brother and me over everything. She quit the tennis team and doesn't see any of her friends." The mother can think of no explanation for these changes. When you interview the patient alone, she becomes angry and says, "Mom should mind her own business and stop talking about me behind my back." She denies use of drugs or alcohol. She tells you, "I don't want to go out of the house because my acne makes me feel ugly. Plus, I've put on ten pounds." Today she weighs 54 kg (120 lb); she is 165 cm (5 ft 5 in) tall. Which of the following is the most appropriate advice to the mother? A) Her daughter is just going through a normal adolescent phase B) Her daughter would benefit from a diet program C) Her daughter would benefit from a trial of antidepressant medication D) She should use a behavior-modification program to control her daughter E) No advice is indicated because it would violate patient confidentiality 35. A 17-year-old African-American girl comes to the health center because she wants contraception. She has been sexually active for the past 2 years. She has never taken oral contraceptive therapy. She says that her boyfriends use condoms some of the time. Past medical history is unremarkable. She takes no medication and has no allergies. She does not drink alcoholic beverages. She smokes one-half pack of cigarettes per day. Her last menstrual period, which was normal, was 3 weeks ago. She has had no vaginal symptoms. The patient is 165 cm (5 ft 5 in) tall and weighs 62 kg (137 lb). Pulse is 62/min and blood pressure is 106/64 mm Hg. Physical examination, including pelvic examination, is normal. Which of the following is the most appropriate recommendation for contraception in this patient? A) Combination oral contraceptive therapy

B) Diaphragm C) Intrauterine device (IUD) D) Levonorgestrel E) Spermicidal jelly 36. A 20-year-old white woman comes to the office because of increasing fatigue for the past 3 weeks. She is 24 weeks pregnant with her first pregnancy. She says she has noted increased difficulty maintaining her usual pace in the local factory where she works packing boxes. Prior to pregnancy, she had no health problems and was considered an above-average worker. Other than the fatigue and an occasional backache, she has had an uneventful pregnancy. Results of initial prenatal laboratory studies were normal, and she has attended all of her prenatal office appointments. Complete blood count is obtained at this visit and a peripheral blood smear, also obtained during this visit, is shown. At this time, the most likely explanation for the patient's fatigue is that she has which of the following? A) Iron-deficiency anemia B) Vitamin B12 (cyanocobalamin) deficiency C) Folate deficiency D) Leukemia E) Physiologic anemia of pregnancy

Block 4 =============================================================== ================= 1. A 75-year-old retired landscape architect is brought to the emergency department after the police found him wandering around the city. He was unable to tell them who he was or where he lived. He has a long history of poorly controlled hypertension. He had two strokes several months ago from which he partially recovered. He can walk but his left arm remains weak. Pertinent family history indicates that his father, a successful businessman, committed suicide at age 72 years. Which of the following is the most likely diagnosis? A) Bipolar disorder, manic episode B) Major depressive disorder C) Vascular dementia D) Dementia, Alzheimer type E) Schizophrenia 2. A 37-year-old woman comes to the emergency department because of a 2-day history of weakness in her left foot which has caused her to accidentally stub the toes of her left foot several times. She reports no other symptoms and is otherwise generally healthy. Medical history is unremarkable except for an episode of blurred vision 1 year ago that resolved on its own. She takes no medications and does not smoke cigarettes or drink

alcoholic beverages. Physical examination shows a left footdrop but is otherwise noncontributory. Neurologic examination shows hyperreflexia of the deep tendon reflexes in all extremities. CT scan of the head shows no abnormalities. Lumbar puncture is done. Examination of this patient's cerebrospinal fluid is most likely to show which of the following? A) Elevated glucose B) Elevated IgG C) Elevated neutrophil count D) Elevated protein E) Positive VDRL 3. A 55-year-old African-American woman is transported to the emergency department by paramedics. She was sitting at the dinner table, and suddenly said to her husband, "I have a terrible headache." She then dropped her fork and slumped in her chair. The husband went to her aid, and she said, "I can't move my right side." He then called emergency medical services. On arrival the patient is alert and oriented. She is holding her head and is in obvious distress. Vital signs are: pulse 101/min, respirations 28/min and blood pressure 190/118z mm Hg. The patient's husband tells you that she has been treated for hypertension for the past 10 years and that she has a 25-year history of simple migraines that occur every 3 to 4 months. On physical examination the patient has an obvious right hemiparesis with an associated hemisensory deficit and hyperreflexia. Blood is drawn for laboratory studies, electrocardiography is done and CT scan of the head is ordered. Her husband asks, "What do you think happened, Doctor?" Based on her clinical presentation and past medical history, which of the following is the most likely cause? A) Cerebellar hemorrhage B) Complicated migraine C) Embolic cerebral infarct D) Intracerebral hemorrhage E) Subarachnoid hemorrhage 4. A 36-year-old firefighter is trapped in a burning building when his air supply runs out. When the rescuers find him 15 minutes later, he is breathing spontaneously and has a pulse, but he is unresponsive. There is no evidence of acute traumatic injury. He is resuscitated with 100% oxygen via a nonrebreathing face mask. He gradually becomes conscious, and by the time he reaches the emergency department, he is awake, alert and complains only of a headache. On arrival in the emergency department arterial blood gas values while breathing 100% oxygen show: PO2 493 mm Hg PCO2 29 mm Hg pH

7.53 Carboxyhemoglobin 30% Which of the following is the most appropriate management at this time? A) Admit the patient for careful monitoring of his airway B) Begin phenytoin therapy by intravenous loading C) Have the patient rebreathe 100% oxygen at 10 L/min D) Intubate the patient in order to allow controlled mechanical ventilation E) Measure the concentration of carbon monoxide in his blood 5. A 23-year-old rugby player is brought to the emergency department by his teammates after sustaining blunt trauma to the left infra-orbital area during a game. The patient complains of significant periorbital pain and says, "I'm seeing flashing lights." Physical examination discloses periorbital swelling. Pupils are equal and reactive. Visual acuity is 20/20 in the right eye and 20/40 in the left eye. Which of the following is the most appropriate step? A) Discharge and follow-up with an ophthalmologist within 24 hours B) Discharge with a nonsteroidal anti-inflammatory drug (NSAID) and an eye patch C) Discharge with corticosteroid eyedrops and a nonsteroidal anti-inflammatory drug (NSAID) D) Discharge with pain medication and application of ice packs E) Immediate consultation with an ophthalmologist 6. A 19-year-old white college student is brought to the emergency department by her two roommates because of shortness of breath. The roommates tell you that her symptoms came on suddenly about one hour after they picked her up at the airport where she had arrived after a 6-hour flight from visiting her parents. The patient has rapid and shallow breathing, and with difficulty she tells you, "I can't get my breath and I'm having pains in my chest. My face is numb. I think I'm dying. Do something. Do something!" She says she has never had an experience like this before. She always has been healthy and she takes no medications except for combination oral contraceptive therapy. Physical examination is normal except for tachypnea and tachycardia. Electrocardiogram shows sinus tachycardia but is otherwise normal. Arterial blood gas values while breathing room air show: PO2 99 mm Hg PCO2 30 mm Hg pH 7.44 Which of the following is the most appropriate long-term pharmacotherapy?

A) Bupropion B) Gabapentin C) Lorazepam D) Metoprolol E) Paroxetine The following vignette applies to the next 2 items. A 58-year-old white man comes to the emergency department because of chest heaviness for the past 45 minutes associated with shortness of breath and a sense of doom. He has not seen a physician since he was 18 years old. He smokes approximately two packs of cigarettes per day. He takes no prescription medications. He has had increasing heartburn recently and he has been taking eight to twelve antacid tablets daily during the past 3 weeks. He is mildly nauseated and diaphoretic. Vital signs are: temperature 37.2C (99.0F), pulse 98/min, respirations 20/min and blood pressure 160/96 mm Hg. Electrocardiogram shows that ST-T segment changes are indeterminate. You are aware of a new blood test, CQ, that can diagnose an acute myocardial infarction (MI) more quickly than the creatine kinase isoenzymes. The receiver operating characteristic (ROC) curve for CQ is shown in the exhibit for four cut-off points. You believe the likelihood of an acute MI is high in this patient and you want to minimize the chance of a false negative. Item 1 of 2 7. Which of the following is the most appropriate cut-off point on the ROC curve? A) A B) B C) C D) D E) It cannot be determined with the data provided Item 2 of 2 8. The cut-off point at which the test performance for CQ is most accurate in the detection of an acute MI is which of the following? A) A B) B C) C D) D E) It cannot be determined from an ROC curve End of Set

9. A 67-year-old man is brought to the emergency department in ventricular fibrillation. His rectal temperature is 26.7C (80.0F). Among the following criteria, the patient should be declared dead when defibrillation fails after which of the following? A) Rapid endotracheal intubation and administration of 100% oxygen B) Restoration of a normal core temperature C) Restoration of a normal oral temperature D) 1 hour of core rewarming E) 20 minutes of warming, externally 10. A 19-year-old white woman is brought to the emergency department by her mother because of intractable emesis. She has a past history of bulimia and according to her mother has been under the care of a psychiatrist. She apparently was well until 1 day ago when she developed emesis and an inability to tolerate liquid and solid foods. Although she takes no medications other than oral contraceptives, she admits to occasional use of both laxatives and ipecac. She denies the use of alcohol or of illicit drugs and says she has experienced no previous symptoms of chest pain, heartburn, hematemesis or fever. Physical examination shows a well-nourished woman with normal vital signs. No lesions are evident on inspection of the skin, but turgor is poor. The thyroid is flat, nontender and without masses. On auscultation of the lungs, moist rales are present at the bases bilaterally. The point of maximal impulse of the heart on the chest wall is 7 cm from the lower left sternal margin in the sixth intercostal space. There is no evidence of an S4 although an S3 is heard. A soft systolic murmur is heard at the apex without a diastolic component. Jugulovenous distention is present 3 cm above the suprasternal notch at 45 degrees' elevation of the chest. The abdomen is soft and a tender liver edge extends 3 cm below the right costal margin. Pitting edema is present in both legs to the mid-calf bilaterally. Laboratory studies show a serum creatinine concentration of 4.2 mg/dL and serum urea nitrogen (BUN) of 88 mg/dL. Urinalysis shows renal tubular epithelial cell casts. Chest x-ray film shows cardiomegaly, central hilar vascular congestion and cephalization of blood flow. Which of the following is the most likely explanation for cardiac decompensation and renal failure in this patient? A) Bulimic cardiomyopathy B) Cocaine intoxication C) Hypothyroidism D) Myocardial ischemia E) Myocarditis 11. A 73-year-old recently widowed African-American woman comes to the emergency department because of abdominal pain, nausea and constipation for the past 3 days. She says the pain has been intermittent and she has had severe nausea and left-sided stomach cramps. She alternates between being constipated and having diarrhea but she has not had a bowel movement in the past 36 hours. She admits to smoking one pack of cigarettes per day, and she has diabetes mellitus that is poorly controlled with diet and glyburide. Vital signs now are: temperature 37.8C (100.8F), pulse 100/min, respirations 28/min and blood pressure 180/90 mm Hg. Abdomen is distended and tender; rectal examination is

positive for occult blood. X-ray film is shown. Which of the following is the most appropriate management at this time? A) Discharge home with clear fluids B) Exploratory laparotomy C) Nasogastric tube D) Oral antibiotics E) Rectal tube 12. A 10-day-old female neonate with Down syndrome is brought to the emergency department by her mother because she has been vomiting for the past 2 days. The vomitus is bile-stained. She was initially breast-fed, but she has been drinking cow milkbased formula for the past week. Physical examination shows slight fullness in the left upper quadrant without obvious tenderness. Test of the stool for occult blood is negative. Which of the following is the most likely diagnosis? A) Allergy to cow milk protein B) Duodenal obstruction C) Hypertrophic pyloric stenosis D) Peptic ulcer with pyloric obstruction E) Small-bowel volvulus The following vignette applies to the next 2 items. A 49-year-old homeless white man comes to the emergency department and says, "I began vomiting 2 to 3 hours ago, and then started to throw up blood." He reports vomiting "about half a cup" of red blood. He had epigastric discomfort after several episodes of emesis, but no preceding abdominal pain. The patient says that he drinks about a half pint of bourbon per day, and he does not use aspirin or nonsteroidal antiinflammatory drugs (NSAIDs). He has no previous history of similar symptoms. Vital signs now are: temperature 37.0C (98.6F), pulse 105/min and blood pressure 150/77 mm Hg. On physical examination he is alert, oriented and disheveled, and he has the odor of alcohol on his breath. No scleral icterus is present. Abdomen is soft, with mild epigastric tenderness. Liver edge is palpated 2 cm below the right costal margin and is nontender. Bowel sounds are present. Stool is negative for occult blood. An intravenous line is started. Endoscopy confirms a tear of the gastroesophageal junction. Laboratory studies show: Serum Amylase 135 U/L BUN 10 mg/dL Creatinine 0.7 mg/dL

Na+ 137 mEq/L K+ 3.3 mEq/L Cl97 mEq/L HCO-3 22 mEq/L Blood Hematocrit 37% Hemoglobin 12 g/dL WBC 12,100/mm3 Platelet count 317,000/mm3 Item 1 of 2 13. The patient says, "What are you going to do, Doc?" Which of the following is the most appropriate next step? A) Cimetidine, intravenously B) Observation and supportive care C) Octreotide, intravenously D) Sclerotherapy E) Selective arterial vasopressin Item 2 of 2 14. After 1 hour the patient's condition has stabilized. Despite your urging him to stay for further evaluation, he insists on leaving. At this time which of the following is most appropriate to tell the patient? A) An elective operation should be scheduled B) He is at immediate risk for major gastrointestinal hemorrhage C) His symptoms might indicate severe alcoholic liver disease D) Omeprazole would help prevent further episodes E) The risk for rebleeding from this episode is relatively small The following vignette applies to the next 2 items.

A 17-year-old African-American boy is brought to the emergency department by his mother and two of his friends at 2:00 AM on Sunday morning. His friends had taken him home from a party after he began to act strangely. They were aware that the patient had recently been treated for marijuana abuse, and they believe that he has significantly cut back on his drug use. His friends began to worry when he insisted that several peers at the party were talking about him behind his back and were plotting to harm him. On the way to the hospital, he was adamant that the same individuals were following them and that they were all in great danger. Now, the patient is unable to walk a straight line, and his speech is slurred. Vital signs are: temperature 38.1C (100.5F), pulse 130/min, respirations 20/min, and blood pressure 150/105 mm Hg. Physical examination discloses vertical and horizontal nystagmus. The remainder of the physical examination and neurologic examination is normal. Item 1 of 2 15. The most likely cause of the patient's current symptoms is intoxication with which of the following? A) Alcohol B) Cannabis C) Heroin D) Nitrous oxide E) Phencyclidine Item 2 of 2 16. Which of the following is the most appropriate pharmacotherapy? A) Diphenhydramine B) Fluoxetine C) Haloperidol D) Hydroxyzine E) Phentolamine The following vignette applies to the next 2 items. A 4-year-old girl is brought to the emergency department by her mother because of severe wrist pain. The girl was playing with her friends in her backyard and fell, breaking the fall with her outstretched hand. On physical examination there is slight swelling over the dorsal aspect of the wrist. X-ray films of the wrist are shown. Item 1 of 2 17. The findings are most consistent with which of the following?

A) Carpal navicular fracture B) Cortical fracture of the radius C) Fracture of the distal radius and ulnar growth plates D) Fracture of the distal radius growth plate E) Perilunate dislocation of the wrist Item 2 of 2 18. Which of the following is the most appropriate management? A) Apply an elastic bandage and apply ice packs to the wrist B) Do closed reduction of the fracture C) Do closed reduction of the fracture and report the case to child protective services D) Immobilize the forearm and hand in situ in a cast E) Prepare for open reduction and internal fixation The following vignette applies to the next 3 items. A 10-year-old boy is brought to the emergency department because he developed hives and shortness of breath 10 minutes after being stung by an insect. His father tells you that he had a similar episode of dyspnea and urticaria 2 years ago. Physical examination now shows a frightened child who appears out of breath, has generalized urticaria and asks for help in a hoarse voice. Vital signs are: temperature 37.0C (98.6F), pulse 120/min, respirations 36/min and blood pressure 70/40 mm Hg. Item 1 of 3 19. Which of the following is the most important first step in managing this patient? A) Administer diphenhydramine, orally B) Administer epinephrine, subcutaneously C) Administer oxygen via face mask D) Establish intravenous access E) Obtain arterial blood gas values Item 2 of 3 20. After 10 minutes there is no change in his condition. At this time, the most appropriate next step is to administer which of the following? A) Diphenhydramine, orally B) Dopamine, intravenously

C) Epinephrine, subcutaneously D) Prednisone, orally E) Ranitidine, orally Item 3 of 3 21. The patient improves with treatment. In 30 minutes his urticaria, dyspnea and hoarseness are resolved and his vital signs are normal. As you prepare to discharge the patient you recommend that he be evaluated by an allergist. The patient's mother asks you what should be done until he can be seen by the allergist. Which of the following is the most appropriate recommendation? A) Avoid all outdoor activity B) Take diphenhydramine, orally, every 4 hours while awake C) Take oral prednisone once daily D) Use an albuterol metered-dose inhaler after any insect sting E) Use an epinephrine autoinjector if he is stung again 22. A 3-year-old girl is brought to the emergency department by her father because of vomiting. He reports that her medical history is unremarkable except for a viral infection 1 month ago, during which she had a mild fever and was irritable for 2 days. She recovered quickly and was well until 3 days ago, when she seemed more thirsty than usual and did not eat as much solid food as she had before. She began vomiting last night and was lethargic today. She is afebrile, pulse is 180/min and respirations are 40/min and deep. On physical examination she is lethargic but responds to touch. Which of the following abnormalities is most likely on further physical examination? A) Acetone-smelling breath B) Hepatomegaly C) Multiple bruises of various stages of healing D) Nuchal rigidity E) An olive-sized mass in the right upper abdominal quadrant 23. A 71-year-old retired oil refinery worker comes to the emergency department at 2:00 AM because of inability to urinate for the past 6 hours. He says he is having abdominal discomfort and that he has had a decreased urinary stream and urinary dribbling for the past 4 months. Vital signs are: temperature 36.5C (97.7F), pulse 103/min and blood pressure 140/90 mm Hg. His lower abdomen is mildly tender and the urinary bladder can be percussed at 2 cm below the umbilicus. Rectal examination shows an enlarged, firm, smooth prostate. Neurologic examination is normal. Which of the following is the most appropriate initial management? A) Admit the patient to the short-stay unit for observation B) Do a suprapubic cystostomy and drain the bladder

C) Insert an indwelling urinary catheter D) Order pelvic ultrasonography E) Order retrograde cystourethrography 24. A 15-year-old girl is brought to the emergency department by her sister. The patient is 36 weeks pregnant and is very upset. She says, "I don't feel the baby move like I used to. Something's wrong!" She has had no prenatal care. A fetal nonstress test is obtained and is nonreactive. Which of the following is the most appropriate first step? A) Assess biophysical profile B) Determine her hemoglobin concentration C) Determine her serum glucose concentration D) Induce labor E) Order amniocentesis to determine fetal maturity The following vignette applies to the next 2 items. A 56-year-old white executive is admitted to the hospital from the emergency department following a severe nosebleed. One month ago he had a brief viral illness after being exposed to an exanthem eruption of one of his grandchildren. At that time the patient was also referred to a urologist because of fatigue, low back pain, and urinary frequency. He was diagnosed with prostatitis, for which he has been taking sulfamethoxazoletrimethoprim for the past 12 days. He does not take any other medications. On arrival in the emergency department vital signs were: temperature 36.8C (98.2F), pulse 100/min, respirations 16/min, and blood pressure 120/66 mm Hg. The patient appeared pale with scattered areas of bruising on his limbs and body and a few petechiae. No lymphadenopathy or organomegaly was found. Results of laboratory studies obtained in the emergency department are shown: Blood Urine Hematocrit 21% WBC 0/hpf Hemoglobin 5.6 g/dL RBC 10-20/hpf WBC 2000/mm3 MCV 102 m3 Partial thromboplastin time 26 sec Platelet count 20,000/mm3 Prothrombin time 12.8 sec INR 1.3 Bone marrow biopsy shows marked hypocellularity. Item 1 of 2 25. Which of the following is the most appropriate management?

A) Administer granulocyte colony-stimulating factor B) Administer high-dose short-term corticosteroids C) Administer parenteral broad-spectrum antibiotics D) Begin transfusion with whole blood E) Discontinue sulfamethoxazole-trimethoprim Item 2 of 2 26. Supportive measures are provided for the patient. Which of the following is the most appropriate treatment recommendation for this patient at this time? A) Chemotherapy B) Follow-up evaluation in 1 week C) Glucocorticoid therapy D) Hematopoietic growth factor therapy E) Stem cell transplant 27. An 87-year-old woman is brought to the emergency department by ambulance. Her friend found her lying in bed in her home about one-half hour ago. She had been incontinent of urine and had also vomited. The patient has a history of degenerative joint disease, hypertension and chronic obstructive pulmonary disease. The paramedics brought in her medications, which include felodipine, naproxen, albuterol inhaler, ipratropium inhaler, prednisone, theophylline and ciprofloxacin. On questioning the woman she says she has a headache and nausea, but she is not able to give a more coherent history. She appears restless, tremulous and agitated. Vital signs are: temperature 37.0C (98.6F), pulse 120/min, respirations 26/min and blood pressure 110/65 mm Hg. Physical examination is normal except for mild expiratory wheezing. Chest x-ray film is normal. Which of the following is the most likely cause of her symptoms? A) Exacerbation of chronic obstructive pulmonary disease B) Gastroenteritis C) Migraine D) Stroke E) Theophylline toxicity 28. A 53-year-old white man is brought to the emergency department by emergency medical services after he crashed his car into a tree. He was not wearing a seatbelt. Upon arrival in the emergency department the patient is clearly drunk but he is cooperative during the examination. Vital signs are: temperature 37.0C (98.6F), pulse 110/min, respirations 18/min and blood pressure 110/75 mm Hg. Physical examination shows generalized tenderness over the lower abdomen and pelvis. Neurologic examination is normal. X-ray films of the cervical spine, chest and pelvis are normal, as is CT scan of

the head. On reexamination 3 hours later, no urinary output has been recorded. The patient is unable to produce a urine sample. He has received 1400 mL of lactated Ringer solution since the accident. Foley catheter is placed and yields 5 mL of bloody urine. Xray film obtained after placement of the Foley catheter is shown. Which of the following is the most appropriate next step? A) Foley catheter drainage for 10 days B) Observation only C) Percutaneous nephrostomy D) Suprapubic catheter drainage E) Surgical repair 29. A 17-year-old white girl is brought to the emergency department after she was struck by a car while riding her bicycle. She was wearing a helmet. She is awake, alert, and oriented. Vital signs are temperature 37.0C (98.6F), pulse 100/min, respirations 18/min, and blood pressure 107/60 mm Hg. Pulse oximetry shows an oxygen saturation of 96% while breathing room air. Physical examination shows no cervical spine tenderness. Breath sounds are clear. Abdominal, pelvic, and neurologic examinations are normal. Screening x-rays of the lateral cervical spine and pelvis are normal. Chest x-ray is shown. Which of the following is the most likely diagnosis? A) Aortic rupture B) Flail chest C) Hemothorax D) Perforated viscus E) Pulmonary contusion The following vignette applies to the next 3 items. You are notified that your patient, a 26-year-old pregnant woman, has been brought by ambulance to the emergency department after she was in an automobile accident. The vehicle in which she was a passenger was broad-sided by another car. She was in the front passenger seat and was wearing a lap/shoulder belt. You have known the patient for 10 years. She is at 34 weeks' gestation with her second pregnancy; she has one child. The nurses attach an external fetal monitor immediately upon the patient's arrival. When you arrive, the patient's vital signs are: pulse 110/min, respirations 18/min and blood pressure 120/80 mm Hg. The fetal heart rate is 150/min with occasional accelerations to 160/min and no decelerations. The monitor shows uterine contractions about every 7 minutes. The patient states that her only discomfort is from the contractions. She says, "They feel like the hard ones from the end of labor with my other baby." On physical examination, the abdomen is very tender to palpation. On speculum examination, there is a small amount of bright red blood oozing from the cervix, which is long and closed. The patient asks you how long she will have to stay in the hospital. Item 1 of 3

30. Which of the following is the most appropriate response at this time? A) "We need to monitor both you and the baby and do some additional tests before I can answer your question." B) "You are in preterm labor from the accident. We will try to stop the contractions with medication, and you can go home later today." C) "You seem a little shaken up, but the baby is fine. I want you to go home but remain in bed the rest of the day." D) "You seem a little shaken up. Even though the baby seems fine, I would like to keep you overnight for observation." E) "You will need to stay here until the baby is delivered." Item 2 of 3 31. While you are talking with the patient, she has a severe contraction that lasts for 5 minutes. Fetal heart tones decrease to 60/min. Which of the following is the most appropriate action at this time? A) Determine fetal scalp pH B) Give the mother oxygen by face mask and magnesium sulfate by slow intravenous push C) Prepare for immediate cesarean delivery D) Place an internal fetal scalp electrode E) Rupture the membranes artificially for vaginal delivery Item 3 of 3 32. The appropriate action is undertaken. The patient asks you if she would have been better off if she had not been wearing a seatbelt. Which of the following is the most appropriate answer? A) "If your car has an air bag, you should not use the seat belt when you are pregnant. But if your car does not have an air bag, we still recommend you use the seat belt." B) "We prefer that pregnant women not wear seat belts because, in case of accidents, the belt can cause more harm than it prevents." C) "Pregnant women should use seat belts until about 28 weeks' gestation. After that, the abdomen is so large that the belt can cause the kind of problem you experienced." D) "The problem you had was caused from the forces of the accident. You might have been injured more seriously without a seat belt." E) "Your problem was that you were sitting in the front passenger seat, which is the most dangerous seat in the car. If you had sat in a different seat, this would not have happened." 33. A 47-year-old man is brought to the emergency department because of the sudden onset of chest pain. On cardiac examination, which of the following physical findings is

suggestive of ischemia? A) Early diastolic murmur at the base B) A late systolic murmur at the apex C) Mid-systolic click D) Pericardial knock E) Pulsus paradoxus 34. A 23-year-old woman comes to the emergency department because of fever. She was diagnosed with acute lymphoblastic leukemia 2 weeks ago, and chemotherapy was initiated 3 days later. She has received all of the chemotherapy as an outpatient. Several hours prior to admission she developed a single episode of shaking chills, which spontaneously resolved. Vital signs are temperature 38.4C (101.1F), pulse 108/min, respirations 16/min, and blood pressure 120/80 mm Hg. She is in mild distress. Partial alopecia is noted, and a Hickman catheter is in place. Results of stat complete blood count are obtained and shown: Blood Hemoglobin 8.5 g/dL WBC 950/mm3 Platelet count 80,000/mm3 Urinalysis shows no abnormalities. Two sets of blood cultures are obtained. Which of the following is the most appropriate next step? A) Administer a single dose of antibiotics and discharge with oral antibiotics B) Admit her to the hospital C) Discharge her on G-CSF (filgrastim) D) Monitor vital signs in the emergency department E) Remove the Hickman catheter and discharge with oral antibiotics The following vignette applies to the next 2 items. An 81-year-old Chinese-American woman is brought to the emergency department by her husband because of back pain. She says, "My back hurts and the pain is getting worse." The patient tells you that 2 days ago, an epidural block was done by a staff anesthesiologist because of a chronic, painful left L5 radiculopathy. She obtained temporary relief immediately after the procedure, but about 24 hours ago she began having midline low back pain without radiation that has increased in severity. She was unsuccessful today in contacting the orthopedist who arranged the procedure. Vital signs now are: temperature 38.5C (101.3F), pulse 101/min and blood pressure 140/85 mm Hg. On physical examination there is tenderness over the L3-5 area in the midline, which is slightly swollen. Straight leg-raising test is negative bilaterally. Anal sphincter tone is normal. There is decreased sensation over the left lateral calf, an absent left ankle reflex

and moderate weakness of left great toe extension. The remainder of the neurologic examination of the leg is normal. Item 1 of 2 35. Which of the following is the most accurate statement? A) An epidural abscess has developed B) The epidural block has worn off C) The original injection was subdural D) A radiculopathy has developed at a different level E) She is having a reaction to the anesthetic Item 2 of 2 36. Which of the following is the most appropriate next step? A) Administer an analgesic and diphenhydramine B) Administer an analgesic, intravenously, and arrange myelography C) Contact the anesthesiologist about repeating the epidural block D) Reassure the patient and have her make an appointment with the orthopedist for tomorrow E) Request an emergency MRI of the spine

Block 5 1. A 76-year-old retired plumber comes to the office with his wife because he has had progressive memory loss during the past year. His wife tells you that he has recently gotten lost in his home at night. He has also had urinary incontinence, about which he says, "It must be because of my big prostate." The patient's wife is concerned that he walks differently and often staggers, but he has not fallen. He now uses a cane. He tells you that he has recently used his wife's lorazepam for insomnia. One of his four siblings has significant memory loss. Vital signs are normal. Physical examination shows good orientation. During the mental status examination, he recalls only one of three items after 3 minutes, he is unable to do serial threes or sevens, and there is evidence of impaired judgment. He has mild ataxia with a tendency to fall to the left if he does not have support. Which of the following is the most likely presumptive diagnosis? A) Dementia, Alzheimer type B) Normal-pressure hydrocephalus C) Parkinson dementia D) Pseudodementia due to depression E) Reversible drug-induced dementia 2. A 52-year-old woman comes to the office because of difficulty falling asleep. She says that she retires to her bedroom at 7:00 PM and watches television while lying on her bed.

She turns out the light and the TV at 11:00 PM but lies awake until at least 2:00 AM. She then sleeps soundly until 7:00 AM when she gets up to go to work. She is divorced and lives alone. She takes no medications. Her physical examination is normal. Which of the following is the most appropriate management? A) Advise her to avoid watching TV in bed B) Advise her to drink a warm beverage before going to bed C) Advise her to exercise lightly at 9:00 PM D) Prescribe flurazepam E) Prescribe temazepam 3. A 7-year-old girl is brought to the office by her mother because the girl has been awakening regularly at night in considerable distress about 1 hour after falling asleep. The mother describes her as being extremely fearful and inconsolable during these episodes. Her mother says, "She cries out something about a figure in a dark cape who is chasing after her and wants to turn her into stone. A few minutes later, she is able to go back to sleep and remembers little of what has happened the next morning." She has been in excellent health and has achieved appropriate developmental milestones. She is doing well in school and has a number of friends and playmates. She is 121 cm (4 ft) tall and weighs 22 kg (48 lb). Vital signs are: temperature 37.0C (98.6F), pulse 60/min, respirations 18/min and blood pressure 100/70 mm Hg. Physical examination shows a normally developed and well-nourished girl. Physical and neurologic examinations are normal. Which of the following is the most likely diagnosis? A) Cataplexy B) Central sleep apnea C) Major depressive disorder D) Nightmares E) Night terrors 4. An 8-year-old boy is brought to the health center by his parents because of a 2-day history of sore throat and fever. Temperature is 38.3C (101.0F) and pulse is 88/min; other vital signs are normal. Physical examination shows erythema of the posterior pharynx; the tonsils are enlarged and there are a few spots of whitish exudate on the left tonsil. A few small, nontender posterior cervical lymph nodes are palpable. There is no rash. Rapid streptococcal test is negative. Which of the following is the most appropriate next step? A) Administer intramuscular penicillin B) Obtain a throat culture C) Obtain acute-phase serum for antistreptolysin-O titer D) Prescribe azithromycin E) Reassure the patient's parents that he has a viral illness 5. A 16-year-old Latino boy comes to the health center because of ear pain for the past 2 days. He has been working at a local restaurant 30 to 40 hours per week through a school-

sponsored vocational education program. His father left his family when the patient was a young child, and his mother died 1 year ago of breast cancer. He was declared an emancipated minor by the court after his mother's death, and he now rents a room in a home in his neighborhood. He has received care at the health center in the past for episodic illnesses and vaccinations before his mother died. Which of the following is the most accurate statement regarding obtaining consent for treatment today? A) Can be seen because he is likely to have an infectious disease and signed consent is not necessary B) Can be seen because his deceased mother gave signed consent for services in the past C) Can be seen if he signs a consent for services D) Cannot be seen because he has a living parent whose signed consent is required E) Cannot be seen without consent of the court that awarded emancipated minor status 6. A 67-year-old man comes to the office because of a 1-week history of increasing cough productive of small amounts of clear sputum and shortness of breath on exertion. He has smoked one and one-half packs of cigarettes per day for the past 40 years. He quit smoking 14 months ago when he was told that he had severe lung disease. He appears to be in no acute distress. He cannot speak in full sentences without taking a breath, and he purses his lips when exhaling. The patient is 170 cm (5 ft 7 in) tall and weighs 57 kg (125 lb); BMI is 20 kg/m2. The physical examination is most likely to show which of the following? A) Accessory muscle use B) An audible, right-sided S3 C) Cyanosis of the extremities D) Lower extremity edema E) Wide splitting of S2 7. A 32-year-old African-American woman with a history of major depressive disorder comes to the health center because of palpitations and dizziness for the past week. She has been taking fluoxetine for the past 3 years and had been doing well until her mother died several months ago. At that time she became despondent with frequent crying and inability to sleep at night. Six weeks ago her psychiatrist increased the dose of fluoxetine but she did not think it helped. A friend recommended St. John's wort, which she began taking several weeks ago. She also takes calcium carbonate daily, occasional antihistamines for seasonal allergies, and amoxicillin-clavulanate, which was prescribed 10 days ago by another physician for a sinus infection. Vital signs now are: temperature 37.2C (99.0F), pulse 90/min, respirations 18/min and blood pressure 140/90 mm Hg. On physical examination she is somewhat tremulous. Thyroid is normal and chest is clear. Cardiovascular examination discloses a regular rhythm with slight tachycardia. Abdominal and neurologic examinations are normal. Which of the following is the most likely cause of her symptoms? A) Anxiety reaction due to her mother's death B) Hyperthyroidism

C) Interaction between fluoxetine and amoxicillin-clavulanate D) Interaction between fluoxetine and antihistamines E) Interaction between fluoxetine and St. John's wort 8. A 54-year-old African-American dispatcher comes to the office because of hip and leg pain. You have treated the patient for diabetes mellitus, emphysema and obesity. His diabetes is controlled with diet and insulin therapy and is managed by his wife who is a registered nurse. He has a 30-year history of smoking two packs of cigarettes per day and he does not want to stop. For the past 4 months he has been taking aspirin with each meal to relieve his hip and leg pain. His wife drops him off at work in the morning; he walks to the hospital to ride home with his wife. He states he has no pain at rest but walking for a few blocks causes his whole left leg to ache. Which of the following is the most likely cause of his symptoms? A) Osteoarthritis B) Peripheral neuropathy C) Peripheral vascular insufficiency D) Sciatic nerve radiculopathy E) Spinal stenosis 9. A 74-year-old woman comes to the office because of constipation and blood-streaked stools for the past 3 days. She has had a 4.5-kg (10-lb) weight loss and anorexia for the past few months. She has a temperature of 37.0C (98.6F). Abdominal examination is normal. Rectal examination is normal except for the presence of occult blood on examination of the stool. Leukocyte count is 9000/mm3. Which of the following is the most likely diagnosis? A) Carcinoma of the cecum B) Carcinoma of the sigmoid colon C) Ischemic colitis D) Pseudomembranous enterocolitis E) Ulcerative colitis 10. A 46-year-old white woman returns to the office for follow-up of abdominal pain. Two weeks ago, she came to the office because of constipation and passing two to three loose stools with mucus per day. Physical examination at that time was normal. Her weight has remained the same and temperature has been normal. She has never had an abdominal operation. Lower gastrointestinal barium study and flexible sigmoidoscopy are normal. Complete blood count is normal. Today, you review the results with the patient. Which of the following is the most appropriate recommendation to the patient? A) Antianxiety medication B) Antispasmodic medication C) Consultation with a gastroenterologist D) Consultation with a psychiatrist E) Fiber supplementation

11. A 10-year-old boy is brought to the office by his mother for a periodic health evaluation. He has been your patient for several years. Recently his teachers and his school counselor have recommended that he be treated with methylphenidate because of his disruptive behavior in the classroom. He has no past history of behavioral problems. Which of the following is the most appropriate advice to the mother? A) "Children with this problem may not have a very high IQ." B) "Children with this problem usually grow out of it by puberty." C) "I don't think he has attention-deficit disorder because he has not had problems in the past." D) "Let's get more information from the school." E) "Methylphenidate treatment will probably improve his grades." 12. A 24-year-old woman comes to the office for an initial prenatal visit. A home pregnancy test was positive 2 weeks ago. Her last menstrual period was 10 weeks ago. She has been healthy and has no significant medical history. She takes no medications. During the interview she becomes tearful and says, "My husband hit me several times 6 weeks ago and I'm afraid that it may happen again. He's become very loving now since he found out about the baby, but I'm still worried." Vital signs now are: temperature 36.9C (98.4F), pulse 80/min, respirations 20/min and blood pressure 110/70 mm Hg. Pelvic examination shows a 10-week size uterus and is otherwise normal. Ultrasonography confirms a 10-week gestation. Which of the following strategies is most appropriate? A) Provide her with the name and phone number of a shelter for battered women B) Reassure the patient that abusive behavior is less likely now that she is pregnant C) Recommend marital counseling D) Schedule an appointment with the husband to discuss his abusive behavior E) Tell her that you will report this to the police if it happens again 13. A 16-year-old boy and his mother come to the office because the mother is concerned that her son may have a drug problem. You have treated him in the past for mild intermittent asthma, and he currently uses an albuterol inhaler prior to exercise. You last saw him 1 year ago for a sports physical examination. At that time, he admitted to drinking alcohol on the weekends. He also admitted to marijuana use but had planned to quit using both before track season started. The mother says that her son now seems disinterested in school and other activities in which he was previously engaged. He has quit the track team, has become irritable, and he has a new set of friends whom his mother dislikes and describes as "druggies." She acknowledges that the boy's father, from whom she is divorced, has an alcohol abuse problem. The boy's older brother abused cocaine for 2 years but is now in a drug rehabilitation program. Prior to interviewing and examining the patient, the mother pulls you aside and asks you to test her son for drugs without informing him. Which of the following is the most appropriate response? A) Agree to do toxicologic screening but review the test results with the patient before giving the results to the mother

B) Agree to do toxicologic screening only if, after discussing it with the patient, he grees to the test C) Agree to do toxicologic screening without the patient's knowledge per his mother's request D) Decline to do toxicologic screening stating that you do not want to get caught between her and her son E) Decline to do toxicologic screening unless under a court order The following vignette applies to the next 2 items. A 75-year-old woman whom you treat for obstructive lung disease comes to the office because of thoracic back pain, which has been troubling her for several weeks. There is no specific history of trauma. Her current medications include ipratropium and inhaled albuterol. She no longer smokes cigarettes and she does not use alcohol. She underwent a mastectomy 10 years ago for breast cancer. Today, vital signs are normal. Her weight is unchanged from 1 year ago. On physical examination breath sounds are diminished in intensity but normal in quality. There is no tenderness over the spinous processes. Laboratory studies in the office show a normal complete blood count and erythrocyte sedimentation rate. A compression fracture of the thoracic spine is seen on a lateral chest x-ray film. After further discussion, the patient agrees to a trial of alendronate to prevent further fractures. Item 1 of 2 14. Regarding alendronate therapy, the patient should be told which of the following? A) The medication should be chewed, rather than swallowed B) The medication should be taken at bedtime C) The medication should be taken with a full glass of water D) The medication should be taken with meals E) She should lie down immediately after taking the medication Item 2 of 2 15. Four days later, the patient returns to the office complaining of chest pain, odynophagia and dysphagia. You decide to discontinue the alendronate. In addition, which of the following is the most appropriate diagnostic study? A) Electrocardiography B) Esophagoscopy C) Laryngoscopy D) MRI of the thoracic spine E) X-ray film of the lateral neck 16. A 69-year-old woman comes to the office because of left knee pain. She says, "For

the past several years my left knee has been bothering me a bit but I have been getting by. However, for the past few days it has been killing me. I was at the outlet mall 2 days ago and I was fine, but yesterday I woke up with such knee pain that I could hardly walk." She denies injuring the knee. No other joints bother her this much although she notes that her right knee occasionally hurts. The pain does not improve with ibuprofen. She has hypertension controlled with hydrochlorothiazide and a -blocking medication, and she has mild chronic renal failure. Serum creatinine concentration 3 months ago was 2.1 mg/dL. She is 168 cm (5 ft 6 in) tall and weighs 106 kg (233 lb). Physical examination today discloses moderate effusion of the left knee. The left knee is warm to touch compared with the right knee. Range of motion is normal but there is crepitus. There is no rash or erythema of the skin. X-ray films of both knees are shown. Arthrocentesis shows: WBC 850/mL Gram stain Negative Microscopic No crystals seen under polarized light Which of the following is the most likely diagnosis? A) Gout B) Osteoarthritis C) Osteoporosis D) Pseudogout E) Septic arthritis 17. A 46-year-old man with diabetes mellitus well controlled by diet alone returns to the office for a follow-up visit for migraines. You have been seeing him for the past year for the migraines, and about 2 months ago you prescribed a calcium-channel-blocking medication. His previous medications included ergotamine and propranolol, but they were discontinued because of lack of full effect. At today's visit he complains of ankle swelling. Physical examination is normal except for 2+ pitting edema. Which of the following is the most appropriate step at this time? A) Determine serum albumin concentration B) Discontinue his calcium-channel-blocking medication C) Order determination of protein excretion in a 24-hour urine sample D) Order echocardiography E) Prescribe a diuretic agent 18. A 27-year-old woman comes to the clinic because of chronic dermatitis involving both ear lobes. She has pierced ears and the rash is most marked around the insertion of her earrings. She should be advised to do which of the following?

A) Avoid skin sensitizing soaps while wearing earrings B) Discontinue the use of metallic earring posts C) Have her ears repierced D) Soak her earrings in 70% alcohol for 24 hours before using them E) Wear clip-on earrings only 19. A 46-year-old woman whom you are treating for hypertension and hypothyroidism returns to the office for a follow-up visit. She denies dyspnea, orthopnea, nocturia or exertional chest discomfort. She states, "Doctor, I've recently found that my feet are swollen. It's becoming hard to get my shoes on. My feet don't really bother me, except that they look funny and make me feel self-conscious." The patient has no other medical problems. She has no allergies. She does not smoke cigarettes and she drinks a glass of wine every evening. Her medications are levothyroxine, extended-release nifedipine and enalapril. Blood pressure is 128/78 mm Hg. On physical examination there are no abnormalities of the skin. Neck veins are not distended. Pulmonary and cardiac examinations are normal. Liver and spleen are not palpable. She has 2+ pitting edema of both legs. The remainder of the examination is normal. Which of the following is the most likely explanation for the edema? A) The patient has deep venous thrombosis B) The patient has misreported her use of alcohol C) The patient has not been taking levothyroxine D) Use of enalapril E) Use of nifedipine 20. A 45-year-old man comes to the office because he has noted a lesion on his chest. He has been generally healthy and you last saw him 5 years ago for an insurance physical examination that was normal. Medical chart notes from that visit report that his skin examination was "normal." You do not recall a lesion being present in the area that he indicates. He is unaware of when the lesion first occurred, but it was noted by a friend at a health club who saw a television report about skin cancer. Physical examination shows a well-appearing man with brown eyes and fair complexion. Vital signs are normal. Skin examination is normal except for the 1-cm lesion on his lateral abdominal wall as shown. The factor that is the best predictor of this patient's prognosis is which of the following? A) Degree of variation in the color of the lesion B) Diameter of the lesion C) His complexion D) His eye color E) Measurement of lesion depth 21. A 34-year-old primigravid woman at 24 weeks' gestation returns to the office to discuss results of a fasting serum glucose study and hemoglobin A1c obtained during a visit 2 weeks ago. Medical history is significant for infertility, oligomenorrhea, and hirsutism. She conceived 3 months after starting treatment with metformin; she

discontinued metformin at 10 weeks' gestation. She currently takes only a prenatal multivitamin supplement. Her mother, a maternal aunt, and a paternal aunt have type 2 diabetes mellitus. The patient is 168 cm (5 ft 6 in) tall and weighs 118 kg (260 lb); BMI is 42 kg/m2. Vital signs are: temperature 37.0C (98.6F), pulse 82/min, respirations 15/min, and blood pressure 112/64 mm Hg. Fasting serum glucose concentration is 120 mg/dL and hemoglobin A1c is 7.5%. Which of the following is the most appropriate management? A) 1200-Calorie American Diabetes Association diet B) Glyburide therapy C) Insulin therapy D) Reinitiating metformin therapy E) Rosiglitazone therapy 22. A 28-year-old African-American woman comes to the office because of low back pain and decreased urine output. She is a semiprofessional basketball player and she is married. She had one pregnancy 12 years ago that was uncomplicated and resulted in vaginal delivery of a term female neonate. Medications include oral contraceptive pills and a corticosteroid inhaler for asthma. She is 182 cm (6 ft) tall and weighs 88 kg (195 lb). Physical examination is normal except for a palpable lower abdominal mass that extends to the umbilicus. Speculum examination discloses a 3-cm ulcerative lesion circumferentially around the external cervical os. Bimanual examination shows a firm, nodular, central pelvic mass filling the pelvis and extending cephalad to the umbilicus. Rectovaginal examination confirms these findings. Stool is negative for occult blood. Which of the following is the most appropriate recommendation? A) Arrange for laparoscopy B) Do a cervical biopsy C) Do an endometrial biopsy D) Increase Pap smear screening to every 3 months E) Obtain pelvic ultrasonography The following vignette applies to the next 3 items. A 61-year-old woman comes to the office because of a 3-month history of urinary incontinence. You have been providing care for her and her husband since they moved to your area 2 years ago. She has a history of diabetes mellitus that was first diagnosed 2 years ago and has been very well managed by diet alone. She currently takes lorazepam at night for sleep and calcium and vitamin D supplements to prevent osteoporosis. She has declined hormone replacement therapy in the past because of a concern about breast cancer. She denies dysuria or problems with incontinence after sneezing or laughing, but she says, "When I try to pass urine, there usually isn't much, but I have to go again a few minutes later. And then, at other times, I just lose control and wet myself. I recently saw a television program on this and I think I have stress incontinence." Vital signs are: temperature 36.9C (98.4F), pulse 64/min, respirations 16/min and blood pressure 158/72 mm Hg. She weighs 93 kg (205 lb) and is 163 cm (5 ft 4 in) tall. Physical

examination is normal except for moderate vaginal mucosal atrophy. Urinalysis done in the office shows no signs of infection. Item 1 of 3 23. Which of the following factors in this patient's history or physical examination suggests a diagnosis other than stress incontinence as the cause for this patient's symptoms? A) Absence of dysuria B) Diabetes mellitus C) Increased body mass index D) Pattern of urination E) Vaginal mucosal atrophy Item 2 of 3 24. The most appropriate next step in management is to suggest which of the following? A) Increased fluid consumption at night B) Referral for bladder ultrasonography C) Replacement of lorazepam with diphenhydramine at night for sleep D) Scheduled voiding E) Use of a pessary Item 3 of 3 25. The patient follows your suggestion. She returns to the office in 3 weeks and reports that she was recently incontinent while shopping with friends. She states, "I've never been so embarassed in my life! Frankly, I don't think I'm better off than when I saw you last time, and now I'm afraid to leave the house." Which of the following is the most appropriate response to the patient's comment? A) "Are you ready to use estrogen therapy now?" B) "Have you ever thought of using adult disposable diapers? They are very effective." C) "I understand your concern. Let's discuss this further and develop a plan." D) "I'm sorry to hear this. I'll refer you to a urologist right away." E) "These things happen. I'm sure you can understand you're growing older." 26. A 27-year-old woman comes to the office because she recently noted a copious vaginal discharge requiring showering or bathing two or three times daily. She states that despite bathing frequently, she never feels clean. She has had no other medical problems. Results of her last Pap smear 1 year ago were normal. She is not sexually active at this time and is taking no medications. She has never been pregnant. She works as a respiratory therapist at a local community hospital and recently broke up with a boyfriend

of several months. Physical examination shows no abnormalities. Pelvic examination demonstrates no vaginal discharge, bleeding, or mucosal lesions. Uterus is normal-sized and nontender. Adnexa are palpable and there are no masses. In addition to obtaining cultures for gonorrhea and chlamydia, which of the following is the most appropriate next step? A) Determine serum estrogen concentration B) Order antibody studies for syphilis and HIV C) Prescribe an oral anti-trichomonal medication and antifungal cream D) Question the patient regarding the circumstances surrounding her recent break-up E) Tell the patient you will wait for the Pap smear results before prescribing anything for the discharge 27. A 77-year-old woman with breast cancer comes to the office because of a 2-week history of severe burning pain and weakness of her left arm. Two months ago, a bone scan obtained because of diffuse bony pain showed widespread metastases. The patient declined chemotherapy and asked for palliative care only. She received localized radiation therapy to left femur and right humerus for the most painful lesions. Her pain was well controlled with celecoxib and a long-acting morphine preparation until 2 weeks ago. Her only other medication is bisacodyl. The patient is alert and oriented. Vital signs are temperature 37.0C (98.6F), pulse 90/min, respirations 20/min, and blood pressure 110/70 mm Hg. She cannot extend her left arm above her head. There is weakness of wrist extension, flexion, and handgrip of the left upper extremity. Sensation to light touch and pinprick is decreased over the left arm. Stroking the left forearm with a cotton swab causes a painful sensation of electric shocks and heat. There are decreased biceps and brachioradialis reflexes on the left. Reflexes, strength, and sensation in the right upper extremity are normal. Neurologic examination of the lower extremities shows no abnormalities. Which of the following is the most appropriate next step in evaluation? A) CT scan of the head B) Electromyography and nerve conduction studies of the left upper extremity C) Measurement of serum B12 (cobalamin) concentration D) MRI of the cervical spine E) Radionuclide bone scan 28. A 32-year-old woman, gravida 3, para 2, who is at 38 weeks' gestation, is brought to the office by her coworker 15 minutes after the patient appeared to have had a seizure at work. The coworker says the patient was working at her computer when she suddenly fell to the floor, began shaking, and had incontinence of urine. She was unresponsive to voice until approximately 5 minutes after the episode. The patient has received routine prenatal care throughout her pregnancy. She has a history of mild, persistent asthma treated with corticosteroid inhalers. Vital signs on arrival are temperature 36.7C (98.0F), pulse 100/min, respirations 22/min, and blood pressure 160/110 mm Hg. The patient is alert but disoriented to time and place. She has no recollection of the episode. Physical examination shows bruising of her left arm and a bite on the lower lip. Which of the following is the most accurate statement regarding the risk of harm to the fetus?

A) The fetus is at risk for developing intrauterine hypoxia B) The fetus will die unless it is delivered immediately C) The risk to the fetus depends on any coexisting respiratory condition D) The risk to the fetus is minimal because the seizure was short-lived E) The risk to the fetus will not be increased if the seizure does not recur 29. A 30-year-old African-American woman returns to the office for a second prenatal visit. She is 12 weeks pregnant and this is her first pregnancy. Results of laboratory studies that were ordered at her first visit show: Blood Hematocrit 28% Hemoglobin 9.2 g/dL Hemoglobin electrophoresis Hemoglobin A1 64% Hemoglobin S 32% Hemoglobin A2 4% Mean corpuscular hemoglobin (MCH) 26 pg/cell Mean corpuscular hemoglobin concentration(MCHC) 32% Hb/cell Mean corpuscular volume (MCV) 74 m3 Which of the following is the most likely cause of her anemia? A) -thalassemia trait B) -thalassemia trait C) Iron deficiency D) Physiologic anemia of pregnancy E) Sickle cell trait 30. A 17-year-old girl brings her 4-day-old neonate to the health center 1 day after discharge from the hospital. She says, "I don't think my baby is getting enough milk. He wants to nurse every 2 hours and my nipples are sore and cracked. I feel miserable." The neonate was born via vaginal delivery without complications, following a normal pregnancy. His birth weight was 3317 g (7 lb 5 oz). Physical examination shows a vigorous, active neonate with a strong sucking reflex. He has a wet diaper on arrival and has had two stools since this morning. There is jaundice of the face. In order to support this new mother during this early stage of breast-feeding, which of the following is the most appropriate advice to the patient? A) Feed the neonate on only one breast at each feeding to ensure complete emptying of the breast B) Insert as much of the areola as possible into the neonate's mouth to improve latching C) Not feed the neonate more than every 3 hours to allow for better milk production D) Offer formula after each feeding to ensure that the neonate is getting enough milk E) Switch to formula for 5 days to assist with healing of her nipples

31. A 19-year-old African-American college student comes to the student health center because of pain in her right knee and fever. She says the pain began about 4 days ago, and she does not recall injuring her knee. Her only medication is an oral contraceptive pill. She is generally healthy. Vital signs are: temperature 37.7C (99.8F), pulse 96/min, respirations 20/min and blood pressure 120/72 mm Hg. She appears uncomfortable. Physical examination is normal except for her right knee, which is red, swollen and tender with a tense effusion. Which of the following tests is most likely to support the diagnosis? A) Cervical cultures B) MRI of the knee C) Plain x-ray film of the knee D) Serum antinuclear antibody titer E) Serum uric acid concentration 32. A 62-year-old woman who was discharged from the hospital several hours ago following evaluation of optic neuritis now has swelling of her ankles. Prior to admission, the patient had a 3-day history of loss of vision and poor color perception in her right eye. Visual acuity on admission was less than 20/200 in her right eye and was 20/40 in her left eye. The remainder of the physical examination was noncontributory. Blood pressure on admission was 140/80 mm Hg. The patient had been otherwise healthy and had been taking no medications. She has never worn corrective lenses. Family history is significant for thyroid disorder. During her hospital stay, the patient received intravenous methylprednisolone therapy for the past four days. Vital signs today are temperature 37.2C (99.0F), pulse 100/min, respirations 14/min, and blood pressure 150/95 mm Hg. Physical examination shows ankle edema but is otherwise unchanged from physical examination done on admission. Laboratory studies are obtained. Which of the following serum laboratory study results is most likely in this patient? A) Decreased magnesium concentration B) Decreased sodium concentration C) Increased calcium concentration D) Increased glucose concentration E) Increased potassium concentration 33. A 30-year-old white woman comes to the office for a periodic health evaluation. At this visit she asks you about a DNA test that she has heard about that can detect the presence of the adult polycystic kidney disease (APKD) gene in asymptomatic carriers. She has a family history of APKD. She has had no symptoms. Her blood pressure is normal, and renal ultrasound 2 months ago was normal. Serum creatinine concentration obtained 2 months ago was 0.9 mg/dL. Before obtaining blood for this test, it is important to explain to this patient that a positive test result would mean which of the following? A) She is certain to develop renal failure B) She may be eligible for disability C) She may have difficulty obtaining life insurance in the future

D) She should avoid becoming pregnant E) She should be monitored for development of liver disease 34. A pharmaceutical representative regularly comes to your office about once a month. Her company has introduced a new prescription nonsteroidal anti-inflammatory drug (NSAID) and she has brought samples today. You overhear your receptionist ask the representative to leave some samples for her own use. If the representative leaves the samples for the receptionist, the most significant concern regarding this situation is that it does which of the following? A) It constitutes practicing medicine without a license B) It diverts samples intended for use by patients C) It may constitute a liability risk D) It promotes the use of a more expensive drug over available alternative medications E) It provides the representative the opportunity to exert undue influence 35. A 54-year-old woman comes to the office for her annual health maintenance examination. At her last visit 11 months ago she reported hot flushes but says they have now resolved. She takes no medications. She is employed as an executive for a computer manufacturing company. Her stress level has increased during the past 9 months because of her company's financial difficulties and she now smokes two packs of cigarettes daily. She is 170 cm (5 ft 7 in) tall and weighs 63 kg (140 lb), which is an increase of 4 kg (9 lb) since her last visit. She plans to begin an intensive aerobics program to stop her weight gain. The patient should be advised that initiation of this exercise program will put her most at risk for which of the following? A) Arthritis B) Exercise-induced asthma C) Gastroesophageal reflux disease D) Stress fractures E) Stroke 36. A 78-year-old retired autoworker from Puerto Rico is brought to the office by his son because of a 4- to 7-kg (9- to 15-lb) weight loss in the past few months. You have been treating this patient for hypertension with hydrochlorothiazide for the past 3 years. Today he reports that he eats only one or two bites of food and feels full and that he has a low energy level. He feels he is just "too old." He has never smoked cigarettes and rarely drinks alcoholic beverages. Which of the following is the most appropriate question to elicit further history that would be helpful in the differential diagnosis of this patient? A) "Are you having headaches?" B) "Did you stop taking your medication?"

C) "Have you had a change in bowel habits?" D) "Have you had any visual changes lately?" E) "What has been going on in your life recently?

S-ar putea să vă placă și